Which of the following numbers are in setA? Check all that apply.
A = {x | x is a positive, odd integer less than 7}
0
1
2
5
3

Answers

Answer 1

Answer:

1, 3, 5.

Step-by-step explanation:

The members of the set are {1, 3, 5}.

Note 0 is an even integer so is not included in the set.

Also 0 is neither negative or positive.


Related Questions

the time taken by a student to the university has been shown to be normally distributed with mean of 16 minutes and standard deviation of 2.1 minutes. He walks in once a day during term time, 180 days per year, and leaves home 20 minutes before his first lecture. a. Find the probability that he is late for his first lecture. b. Find the number of days per year he is likely to be late for his first lecture.

Answers

Answer:

a) 2.84% probability that he is late for his first lecture.

b) 5.112 days

Step-by-step explanation:

When the distribution is normal, we use the z-score formula.

In a set with mean [tex]\mu[/tex] and standard deviation [tex]\sigma[/tex], the zscore of a measure X is given by:

[tex]Z = \frac{X - \mu}{\sigma}[/tex]

The Z-score measures how many standard deviations the measure is from the mean. After finding the Z-score, we look at the z-score table and find the p-value associated with this z-score. This p-value is the probability that the value of the measure is smaller than X, that is, the percentile of X. Subtracting 1 by the pvalue, we get the probability that the value of the measure is greater than X.

In this question, we have that:

[tex]\mu = 16, \sigma = 2.1[/tex]

a. Find the probability that he is late for his first lecture.

This is the probability that he takes more than 20 minutes to walk, which is 1 subtracted by the pvalue of Z when X = 20. So

[tex]Z = \frac{X - \mu}{\sigma}[/tex]

[tex]Z = \frac{20 - 16}{2.1}[/tex]

[tex]Z = 1.905[/tex]

[tex]Z = 1.905[/tex] has a pvalue of 0.9716

1 - 0.9716 = 0.0284

2.84% probability that he is late for his first lecture.

b. Find the number of days per year he is likely to be late for his first lecture.

Each day, 2.84% probability that he is late for his first lecture.

Out of 180

0.0284*180 = 5.112 days

g A psychic was tested for extrasensory perception (ESP). The psychic was presented with cards face down and asked to determine if each of the cards was one of four symbols: a star, cross, circle, or square. Let p represent the probability that the psychic correctly identified the symbols on the cards in a random trial. How large a sample n would you need to estimate p with margin of error 0.01 and 95% confidence?

Answers

Answer:

Step-by-step explanation:

Hello!

The objective is to test ESP, for this, a psychic was presented with cards face down and asked to determine if each of the cards was one of four symbols: a star, cross, circle, square.

Be X: number of times the psychic identifies the symbols on the cards correctly is a size n sample.

p the probability that the psychic identified the symbol on the cards correctly

You have to calculate the sample size n to estimate the proportion with a confidence level of 95% and a margin of error of d=0.01

The CI for the population proportion is constructed "sample proportion" ± "margin of error" Symbolically:

p' ± [tex]Z_{1-\alpha /2} * (\sqrt{\frac{p'(1-p')}{n} } )[/tex]

Where  [tex]d= Z_{1-\alpha /2} * (\sqrt{\frac{p'(1-p')}{n} } )[/tex] is the margin of error. As you can see, the formula contains the sample proportion (it is normally symbolized p-hat, in this explanation I'll continue to symbolize it p'), you have to do the following consideration:

Every time the psychic has to identify a card he can make two choices:

"Success" he identifies the card correctly

"Failure" he does not identify the card correctly

If we assume that each symbol has the same probability of being chosen at random P(star)=P(cross)=P(circle)=P(square)= 1/4= 0.25

Let's say, for example, that the card has the star symbol.

The probability of identifying it correctly will be P(success)= P(star)= 1/4= 0.25

And the probability of not identifying it correctly will be P(failure)= P(cross) + P(circle) + P(square)= 1/4 + 1/4 + 1/4= 3/4= 0.75

So for this experiment, we'll assume the "worst case scenario" and use p'= 1/4 as the estimated probability of the psychic identifying the symbol on the card correctly.

The value of Z will be [tex]Z_{1-\alpha /2}= Z_{0.975}= 1.96[/tex]

Now using the formula you have to clear the sample size:

[tex]d= Z_{1-\alpha /2} * (\sqrt{\frac{p'(1-p')}{n} } )[/tex]

[tex]\frac{d}{Z_{1-\alpha /2}} = \sqrt{\frac{p'(1-p')}{n} }[/tex]

[tex](\frac{d}{Z_{1-\alpha /2}})^2 =\frac{p'(1-p')}{n}[/tex]

[tex]n*(\frac{d}{Z_{1-\alpha /2}})^2 = p'(1-p')[/tex]

[tex]n = p'(1-p')*(\frac{Z_{1-\alpha /2}}{d})^2[/tex]

[tex]n = (0.25*0.75)*(\frac{1.96}{0.01})^2= 7203[/tex]

To estimate p with a margin of error of 0.01 and a 95% confidence level you have to take a sample of 7203 cards.

I hope this helps!

Answer:

The sample size should be 6157

Step-by-step explanation:

Given that the margin of error (e) = ± 0.01 and the confidence (C) = 95% = 0.95.

Let us assume that the guess p = 0.25 as the value of p.

α = 1 - C = 1 - 0.95 = 0.05

[tex]\frac{\alpha }{2} =\frac{0.05}{2}=0.025[/tex]

The Z score of α/2 is the same as the z score of 0.475 (0.5 - 0.025) which is 1.96. Therefore [tex]Z_\frac{\alpha }{2}=Z_{0.025}=1.96[/tex]

To determine the sample size n, we use the formula:

[tex]Z_{0.025}*\sqrt{\frac{p(1-p)}{n} }\leq e\\Substituting:\\1.96*\sqrt{\frac{0.2(1-0.2)}{n} } \leq 0.01\\\sqrt{\frac{0.2(0.8)}{n} }\leq \frac{1}{196}\\\sqrt{0.16} *196 \leq \sqrt{n}\\78.4\leq \sqrt{n}\\ 6146.56\leq n\\n=6157[/tex]

FIND P(NOT 6) WHEN YOU ROLL A STANDARD NUMBER CUBE THEN DESCRIBE THE LIKELIHOOD OF THE EVENT WRITE IMPOSSIBLE ,UNLIKELY , EQUALLY LIKELY , LIKLEY OR CERAIN

Answers

Answer: LIKLEY

Step-by-step explanation:

Formula : Probability [tex]=\dfrac{\text{Number of favorable outcomes}}{\text{Total outcomes}}[/tex]

A standard cube has six numbers on it (1,2,3,4,5 and 6).

P( NOT 6) =[tex]\dfrac{\text{Numbers that are not 6}}{\text{Total numbers}}[/tex]

[tex]=\dfrac{5}{6}=0.8333[/tex]

We know that when the probability of any event lies between 0.5 and 1then the event is said to be likely to happen.

Since , P(not 6)=0.8333 which lies between 0 and 0.5.

That means, it is likely to happen.

Note :

When probability of having A = 0 , we call A as uncertain event.

When probability of having A = 1 , we call A as certain event.

When probability of having A = 0.5 , we call A as equally likely event.

When probability of having A lies between 0 and 0.5 , we call A as unlikely event.

When probability of having A lies between 0.5 and 1 , we call A as likely event.

Skyler is out shopping and sees that striped shirts are on sale for
$19.00 each, and plaid pants are on sale for $19.50 each. He
buys 8 shirts and 6 pairs of pants. What is the total of his
purchase?

The total was $_______​

Answers

Answer:

His total is $269

Step-by-step explanation:

8x19 = 152

6x19.50 = 117

152+117 = 269

Which ordered pair is the solution of the system of equations? 3x+2y=4, -2+2y=24

Answers

Answer:

x = -7.33        OR         x = [tex]\frac{-22}{3}[/tex]

y = 13

Step-by-step explanation:

→You can use the substitution method. First, make y by itself in (-2 + 2y = 24):

-2 + 2y = 24

2y = 26

y = 13

→Then, plug in 13 for y into the other equation:

3x + 2y = 4

3x + 2(13) = 4

3x + 26 = 4

3x = -22

x = -7.33        OR         x = [tex]\frac{-22}{3}[/tex]

What’s the correct answer for this question?

Answers

Answer:

D

Step-by-step explanation:

The volume of pyramid = 1/3 wlh

Where w = width, l = length and h = height

While,

The volume of rectangular prism = wlh

So,

The volume of pyramid = 1/3(the volume of prism)

Researchers recorded that a certain bacteria population declined from 750,000 to 250 in 48 hours after the administration of medication. At this rate of decay, how many bacteria will there be in 8 hours?

Answers

Answer:

There will be 66 bacteria in 8 hours.

Step-by-step explanation:

The number of bacteria after t hours is given by the following formula.

[tex]P(t) = P(0)(1-r)^{t}[/tex]

In which P(0) is the initual number of bacteria and r is the decay rate.

Researchers recorded that a certain bacteria population declined from 750,000 to 250 in 48 hours after the administration of medication.

This means that [tex]P(0) = 750000, P(48) = 250[/tex]

We use this to find r. So

[tex]P(t) = P(0)(1-r)^{t}[/tex]

[tex]250 = 750000(1-r)^{48}[/tex]

[tex](1-r)^{48} = \frac{250}{750000}[/tex]

[tex]\sqrt[48]{(1-r)^{48}} = \sqrt[48]{\frac{250}{750000}}[/tex]

[tex]1-r = 0.84637[/tex]

So

[tex]P(t) = 750000(0.84637)^{t}[/tex]

How many bacteria will there be in 8 hours?

8 hours from now, in this context, is 8 + 48 = 56 hours. So this is P(56).

[tex]P(56) = 750000(0.84637)^{56} = 65.83[/tex]

Rounding to the nearest number

There will be 66 bacteria in 8 hours.

Answer:

197,488

Step-by-step explanation:

This problem requires two main steps. First, we must find the unknown rate, k. Then, we use that value of k to help us find the unknown number of bacteria.

Identify the variables in the formula.

AA0ktA=250=750,000=?=48hours=A0ekt

Substitute the values in the formula.

250=750,000ek⋅48

Solve for k. Divide each side by 750,000.

13,000=e48k

Take the natural log of each side.

ln13,000=lne48k

Use the power property.

ln13,000=48klne

Simplify.

ln13,000=48k

Divide each side by 48.

ln13,00048=k

Approximate the answer.

k≈−0.167

We use this rate of growth to predict the number of bacteria there will be in 8 hours.

AA0ktA=?=750,000=ln13,00048=8hours=A0ekt

Substitute in the values.

A=750,000eln13,00048⋅8

Evaluate.

A≈197,488.16

At this rate of decay, researchers can expect 197,488 bacteria.

If 6 newborn babies are randomly selected, how many different gender sequences are possible?

Answers

Answer:

720

Step-by-step explanation:

6!

6x5x4x3x2x1=720

A digital camcorder repair service has set a goal not to exceed an average of 5 working days from the time the unit is brought in to the time repairs are completed. A random sample of 12 repair records showed the following repair times (in days): 5, 7, 4, 6, 7, 5, 5, 6, 4, 4, 7, 5.
H0: \mu \leq 5 days versus H1: \mu > 5 days. At \alpha = .05, choose the right option.
a) Reject H0 if tcalc < 1.7960
b) Reject H0 if tcalc >1.7960

Answers

Answer:

The degrees of freedom first given by:  

[tex]df=n-1=12-1=11[/tex]  

Then we can find the critical value taking in count the degrees of freedom and the alternative hypothesis and then we need to find a critical value who accumulates 0.05 of the area in the right tail and we got:

[tex] t_{\alpha}= 1.796[/tex]

And for this case the rejection region would be:

b) Reject H0 if tcalc >1.7960

Step-by-step explanation:

Information given

5, 7, 4, 6, 7, 5, 5, 6, 4, 4, 7, 5.

System of hypothesis

We want to test if the true mean is higher than 5, the system of hypothesis are :  

Null hypothesis:[tex]\mu \leq 5[/tex]  

Alternative hypothesis:[tex]\mu > 5[/tex]  

The statistic is given by:

[tex]t=\frac{\bar X-\mu_o}{\frac{s}{\sqrt{n}}}[/tex] (1)  

The degrees of freedom first given by:  

[tex]df=n-1=12-1=11[/tex]  

Then we can find the critical value taking in count the degrees of freedom and the alternative hypothesis and then we need to find a critical value who accumulates 0.05 of the area in the right tail and we got:

[tex] t_{\alpha}= 1.796[/tex]

And for this case the rejection region would be:

b) Reject H0 if tcalc >1.7960

Find the amount to which $2,500 will grow if interest of 6.75% is compounded quarterly for 10
years.



Find the amount to which $2,500 will grow if interest of 6.75% is compounded daily for 10
years.

Answers

Answer:

Part a

For this case n = 4. If we use the future value formula we got:

[tex] A= 2500 (1+ \frac{0.0675}{4})^{4*10}= 4882.506[/tex]

Part b

For this case n = 365. If we use the future value formula we got:

[tex] A= 2500 (1+ \frac{0.0675}{365})^{365*10}= 4909.776[/tex]

Step-by-step explanation:

We can use the future vaue formula for compound interest given by:

[tex] A= P(1+ \frac{r}{n})^{nt}[/tex]

Where P represent the present value, r=0.0675 , n is the number of times that the interest is compounded in a year and t the number of years.

Part a

For this case n = 4. If we use the future value formula we got:

[tex] A= 2500 (1+ \frac{0.0675}{4})^{4*10}= 4882.506[/tex]

Part b

For this case n = 365. If we use the future value formula we got:

[tex] A= 2500 (1+ \frac{0.0675}{365})^{365*10}= 4909.776[/tex]

Find the area of the triangle

Answers

Answer:

Step-by-step explanation:

The formula for the area of a triangle is base*height divided by 2. Remember this because itll be important for everything you do in math relating to geometry and calculus. Assuming you go that far

[tex]\frac{base*height}{2} =\frac{14*8}{2} =\frac{112}{2} = 56 units^2[/tex]

Answer:

A =56 units^2

Step-by-step explanation:

The area of a triangle is given by

A =1/2 bh where 14 is the base and 8 is the height

A = 1/2 (14)8

A =56 units^2

In a particular region, for families with a combined income of $75,000 or more, 15% of these families have no children, 35% of the families have one child, 45% have two children, and 5% have three children. Use this information to construct the probability distribution for X, where x represents the number of children per family for this income group. Arrange x in increasing order and write the probabilities P(x) as decimals

Answers

Answer:

The probability distribution for x:"number of children per family for this income group" is:

[tex]\text{P(x=0)}=0.15\\\\\text{P(x=1)}=0.35\\\\\text{P(x=2)}=0.45\\\\\text{P(x=3)}=0.05\\\\[/tex]

Step-by-step explanation:

With the information given we have the relative frequencies of each category.

We know:

[tex]\text{P(x=0)}=0.15\\\\\text{P(x=1)}=0.35\\\\\text{P(x=2)}=0.45\\\\\text{P(x=3)}=0.05\\\\[/tex]

Which number is irrational

Answers

Answer:

Can you give the question. Can you post the picture. I can help solve. I will edit this answer once you have given the question/picture.

is 7.68 bigger than 7.680

Answers

Answer:

literally 7.68=7.680

Help asap giving branlist!!!

Answers

Answer:

D.

Step-by-step explanation:

So you know you have to have $62 as the base fee.

If you exceed 2 gigabytes, you subtract that by 2 because you want to find how many gigabytes you're going over. You then multiply it by 30 to find the cost.

You get C = 62 + 30(g - 2)

Answer:

anwser is d because it is write.

Step-by-step explanation:

What is the missing side length?

Answers

Answer:

8 yds

Step-by-step explanation:

The sides have to have the same length

14 yd = 6yd + ?

Subtract 6 from each side

14-6 = 8

8 yds

The soup produced by a company has a salt level that is normally distributed with a mean of 5.4 grams and a standard deviation of 0.3 grams. The company takes readings of every 10th bar off the production line. The reading points are 5.8, 5.9, 4.9, 5.2, 5.0, 4.9, 6.2, 5.1, 5.7, 6.1. Is the process in control or out of control and why?

Answers

Answer:

Step-by-step explanation:

The mean of the reading points is

Mean = (5.8 + 5.9 + 4.9 + 5.2 + 5.0 + 4.9 + 6.2 + 5.1 + 5.7 + 6.1)/10 = 5.48

The process is out of control if the mean salt level of the readings is greater than 5.4

For the null hypothesis,

µ = 5.4

For the alternative hypothesis,

µ > 5.4

This is a right tailed test.

Since the population standard deviation is given, z score would be determined from the normal distribution table. The formula is

z = (x - µ)/(σ/√n)

Where

x = sample mean

µ = population mean

σ = population standard deviation

n = number of samples

From the information given,

µ = 5.4

x = 5.48

σ = 0.3

n = 10

z = (5.48 - 5.4)/(0.3/√10) = 0.84

Looking at the normal distribution table, the probability corresponding to the z score is 0.7996

The probability value to the right of the z score is 1 - 0.7996 = 0.2

Assuming a significance level of 0.05

Since alpha, 0.05 < than the p value, 0.2, then we would fail to reject the null hypothesis. Therefore, At a 5% level of significance, we can conclude that the process is not out of control. If we had rejected the null hypothesis, then our conclusion would be that the process is out of control.

Please answer this correctly I have to finish this today as this is my deadline

Answers

Answer:

r = 1.499619733762 m  There is no such thing a quarter radius!

C = 9.4223886775301 m

A = 7.065 m^2

Step-by-step explanation:

Calculate r and C | Given A

Given the area of a circle calculate the radius and circumference

r = √(A / π)

C = 2πr

Agenda:

r = radius

C = circumference

A = area

π = pi = 3.1415926535898

√ = square root

What is the volume of the rectangular prism?

Answers

Answer:

10ft[tex]{3}[/tex]

Step-by-step explanation:

One face has 15 blocks of 1/3 ft. You can clearly see 2 sets of blocks.

15 x 2 = 30

30 ÷ 3 or 30 x 1/3

= 10 ft cubed

In order to understand reasons why consumers visit their store, a local business conducts a survey by asking the next 100 people who visit their store to fill out a short survey. The business finds that 40 of the 100 people state that the main reason they visited the store was because the store is running a sale on coats that week. A confidence interval is constructed for the population proportion of consumers who would visit the store because of the coat sale. Which confidence interval would be the narrowest?

a. 90%
b. 99%
c. 95%
d. 85%

Answers

Answer:

d. 85%

Step-by-step explanation:

In a sample with a number n of people surveyed with a probability of a success of [tex]\pi[/tex], and a confidence level of [tex]1-\alpha[/tex], we have the following confidence interval of proportions.

[tex]\pi \pm z\sqrt{\frac{\pi(1-\pi)}{n}}[/tex]

In which

z is the zscore that has a pvalue of [tex]1 - \frac{\alpha}{2}[/tex].

The margin of error is:

[tex]M = z\sqrt{\frac{\pi(1-\pi)}{n}}[/tex]

The higher the confidence level, the higher the value of z, which means that the margin of error will be higher and the interval will be wider,

Which confidence interval would be the narrowest?

The one with the lowest confidence level. So the answer is d.

Write an equation of a line that passes through (-6, 1), parallel to y = 2x – 6.

Answers

Answer:

y = -1/2x - 2

Step-by-step explanation:

If it's parallel, that means that the slope is the opposite of the one in the given equation, meaning that 2 would be flipped and turned negative into -1/2.

Then, fill in the x and y values to get the y-intercept.

1 = -1/2(-6) + b

1 = 3 + b

-2 = b

So your answer is y = -1/2x - 2

Please answer this correctly

Answers

Answer:

[tex]h=\sqrt{1.44}\\h = 1.2[/tex]

Step-by-step explanation:

Base of the triangle on the left = 0.5

Use pythagorean theorem

[tex]a^{2} + b^{2} = c^{2}[/tex]

Substitute

[tex]0.5^{2} + b^{2} = 1.3^{2}[/tex]

[tex]b^{2} = 1.3^2 - 0.5^2[/tex]

[tex]b^2 = 1.44[/tex]

[tex]b = \sqrt{1.44} \\[/tex]

[tex]b = 1.2[/tex]

in this case b is the height

so

[tex]h=\sqrt{1.44}\\h = 1.2[/tex]

The mean height of women in a country​ (ages 20minus​29) is 64.2 inches. A random sample of 75 women in this age group is selected. What is the probability that the mean height for the sample is greater than 65 ​inches? Assume sigmaequals2.84. The probability that the mean height for the sample is greater than 65 inches is nothing.

Answers

Answer:

[tex] z=\frac{65-64.2}{\frac{2.84}{\sqrt{75}}} = 2.440[/tex]

And we can find the probability using the complement rule and with the normal standard table like this:

[tex] P(Z>2.440) =1-P(Z<2.440) = 1-0.993 =0.007[/tex]

The probability that the mean height for the sample is greater than 65 inches is 0.007

Step-by-step explanation:

Let X the random variable that represent the women heights of a population, and we know the following parameters

[tex]\mu=64.2[/tex] and [tex]\sigma=2.84[/tex]

We are interested on this probability

[tex]P(X>65)[/tex]

Since the sample size selected is 75>30 we can use the centrel limit theorem and the appropiate formula to use would be the z score given by:

[tex]z=\frac{x-\mu}{\frac{\sigma}{\sqrt{n}}}[/tex]

If we find the z score for 65 inches we got:

[tex] z=\frac{65-64.2}{\frac{2.84}{\sqrt{75}}} = 2.440[/tex]

And we can find the probability using the complement rule and with the normal standard table like this:

[tex] P(Z>2.440) =1-P(Z<2.440) = 1-0.993 =0.007[/tex]

The probability that the mean height for the sample is greater than 65 inches is 0.007

80 81 82 83 84 85 86 87 88 89 90

Anika's test scores are shown below.

Anika's Test Scores

80 81 82 83 84 85 86 87 88 89 90

Which statement compares the shape of the two dot plots?

There is a gap in both plots.

There is a gap in Anika's scores, but not in Lorenzo's scores.

The data is widely spread across both plots.

The data is more widely spread for Lorenzo's scores than for Anika's.

Mark this and return

Save and Exit

Answers

Answer:

D :)

Step-by-step explanation:

Answer:

D

Step-by-step explanation:

Braily please

1/216^-2/3 + 1/256^-3/4 + 1/243^-1/5​

Answers

Answer:

103

Step-by-step explanation:

[tex]\dfrac{1}{216}^{-2/3}+\dfrac{1}{256}^{-3/4}+\dfrac{1}{243}^{-1/5}= \\\\\\\sqrt[3]{216^2}+\sqrt[4]{256^3}+\sqrt[5]{243}=\\\\\\6^2+4^3+3=\\\\\\36+64+3=\\\\\\103[/tex]

Hope this helps!

What is the difference of the polynomials? (–2x3y2 + 4x2y3 – 3xy4) – (6x4y – 5x2y3 – y5)

Answers

Answer:

-6x⁴y - 2x³y² + 9x²y³ - 3xy⁴ + y⁵

Step-by-step explanation:

(–2x³y² + 4x²y³ – 3xy⁴) – (6x⁴y – 5x²y³ – y⁵)=

–2x³y² + 4x²y³ – 3xy⁴ – 6x⁴y + 5x²y³ + y⁵=

-6x⁴y - 2x³y² + 9x²y³ - 3xy⁴ + y⁵

Ralph is 3 times as old as Sara. In 4 years, Ralph will be only tice as old as Sara will be then.
If x represents Sara's age now, which of the following expressions represents Ralph's age in four years?

A. 3x
B. 2x+4
C. 3x+4

Answers

Answer:

In 6 years, Ralph will be only twice as old as Sara

Step-by-step explanation:

Answer:

The answer is C, 3x+4

Step-by-step explanation:

The “in four years” part translates to +4. The 3x translates to 3 times his current age. Hope this helped :)

An appliance repairman charges $25 plus $40 per hour for house calls. Write the rule as an equation that relates hours worked x and his fee y.

Answers

To get the total fee, you need to multiply the hourly rate by number of hours worked and add that to the flat fee of $25.

The equation would be y = 40x + 25

ASAP! GIVING BRAINLIEST! Please read the question THEN answer CORRECTLY! NO guessing. I say no guessing because people usually guess on my questions.

Answers

Answer: f(x)=2-x^2

Step-by-step explanation:

The quadratic equation is

y=ax^2+bx+c

and c is equal to the y-intercept.

in the twi graphs shown both have the same shape but different y-intervepts.

c(the y-intercept) in the first graph is 5 and in the second graph(F) is 2.

On the graphing calculator it says that f(x)=2-x^2 is the correct answer therefore it is correct.

Which set of ordered pairs does NOT represent a function ?

Answers

Answer:

The answer is C.

Step-by-step explanation:

For a function, we do a vertical line test. If there is more than one point in one single x-position, it is not a function. Example, the ordered pairs (1, 1) and (1, 2) do NOT describe a function because there are more than one point on x=1.

Other Questions
. If the U.S. enacted an educational system like that of Sparta would it have a negative or positive impact on the school systems? How? Researchers should generally avoid primary sources because truth can be lost when an idea has been interpreted by two or three people.( True False ) For the graph, locate the x-intercept and y-intercept. x-intercept = -4, 4, -8, or 8y-intercept = -4, 4, -8, or 8 According to this case study, what is an upcoming key technology that will be used in retail stores to improve customer service? And how it is currently being used? What will be the role of smartphones in the future of shopping? Support your claim with a reference. HLP ! HLP ! HLP ! Last one what is deplasmolysis Me. Parsons wants to build a new banister for the staircase shown. If the rise of the stairs of a building is 5 feet and the run is 12 feet, what will be the length of the new banister? I need this to be explained Poor weather would not allow Odysseus and his men to Find the value of x. If necessary, round to the nearest tenth What is mzABC?75Om ABC = 15Om ABC = 45135Om ABC = 60Om ABC = 75BD PLEASE ANSWERIn a dysfunctional family children are socialised and taught the acceptable norms and values of society???? True or False???? Design and complete a frequency table for Belinda.Belinda ask 20 people, how many hours of TV did you watch last week?Here is the results3,17,4,4,6,11,14,14,1,20,9,8,9,6,12,7,8,13,13,9.Belinda wants to show these result in a frequency table.She will use 4 equal groups.The first group will start with 1 hour and the last group will end with 20 hours. What was the main cause for the end of Reconstruction?O A. the economic crisis meant that Democrats could not conpay for Reconstruction.B. Democrats had gained enough power to make a change.C. The military situation made it dangerous for the Republicarcontinue their policies.D. Republicans were eager to move on to other issues. HELP I DONT GET THIS AT ALL. ITS ABOUT INEQUALITIES EXPLANATION NEEDED 2 PointsWorld War I propaganda referred to Germans as "Huns" to make them seem:A. BarbaricB. SillyC. StrongD. UntrustworthySUBMIT Identify the characterization technique used in the following statements and then write a sentence using the opposite kind of characterization. The first has been done for you.1.She wore a ten carat diamond necklace everywhere she went.Indirect characterization. Direct:She was rich.2.Joe was having fun at the party._______________________________________________________________________________________________________________________________________3.All Sally does when people are around is say weird things that nobody understands._______________________________________________________________________________________________________________________________________ During a reaction, the enthalpy of the reactants and the products are not equal. What does this result in?A. None of theseB. Absorption or release of energyC. Change in chemical compositionD. Spin of electrons is reversed name of main component of fifth generation of computer If y is the number of yellow cars which variable expression represents the phrase below? The sum of the number of yellow cars and 9 red cars Michael has a home-based business putting on children's parties. He charges $45 to design the party and then $7.50 per child . Write a function rule ( equation) that relates the total cost of the party to the number of children n